Difference between revisions of "1980 AHSME Problems/Problem 24"
(Created page with "== Problem == For some real number <math>r</math>, the polynomial <math>8x^3-4x^2-42x+45</math> is divisible by <math>(x-r)^2</math>. Which of the following numbers is closest t...") |
J314andrews (talk | contribs) |
||
(3 intermediate revisions by 2 users not shown) | |||
Line 9: | Line 9: | ||
\text{(E)} \ 1.62 </math> | \text{(E)} \ 1.62 </math> | ||
− | == Solution == | + | == Solution 1 == |
− | <math>\ | + | |
+ | Since <math>(x-r)^2</math> is a factor of <math>8x^3-4x^2-42x+45</math>, <math>r</math> is a double root. Let <math>s</math> be the third root of <math>8x^3-4x^2-42x+45</math>. | ||
+ | |||
+ | By Vieta's formulas, <math>2r+s = \frac{1}{2}</math>, so <math>s = \frac{1}{2} - 2r</math>. | ||
+ | |||
+ | Also, by Vieta's formulas, <math>r^2 + 2rs = -\frac{21}{4}</math>. Substituting <math>s = \frac{1}{2} - 2r</math> yields <math>r^2 + 2r\left(\frac{1}{2} - 2r\right) = -3r^2 + r = -\frac{21}{4}</math>. So <math>12r^2 - 4r - 21 = 0</math>, that is, <math>(6r + 7)(2r - 3) = 0</math>. Therefore, <math>(r, s)</math> is either <math>\left(\frac{3}{2}, -\frac{5}{2}\right)</math> or <math>\left(-\frac{7}{6}, \frac{17}{6}\right)</math>. | ||
+ | |||
+ | Finally, by Vieta's formulas, <math>r^2s = -\frac{45}{8}</math>. Though <math>(r, s) = \left(-\frac{7}{6}, \frac{17}{6}\right)</math> does not satisfy this equation, <math>\left(r, s\right) = \left(\frac{3}{2}, -\frac{5}{2}\right)</math> does. | ||
+ | |||
+ | Therefore, <math>r = \frac{3}{2} = 1.5</math>, so <math>\boxed{\text{(\textbf{D})} \ 1.52}</math> is closest to <math>r</math>. | ||
+ | |||
+ | -e_power_pi_times_i, edited by j314andrews | ||
+ | |||
+ | == Solution 2 == | ||
+ | |||
+ | By polynomial long division, <math>\frac{8x^3 - 4x^2 - 42x + 45}{x^2 - 2rx + r^2} = 8x + (16r-4) + \frac{(24r^2 - 8r - 42)x + (-16r^3 + 4r^2 + 45)}{x^2 - 2rx + r^2}</math>. | ||
+ | |||
+ | So <math>(24r^2 - 8r - 42)x + (-16r^3 + 4r^2 + 45) = 0</math>, that is, <math>r</math> is a root of both <math>24r^2 - 8r - 42</math> and <math>-16r^3 + 4r^2 + 45</math>. | ||
+ | |||
+ | Since <math>24r^2 - 8r - 42 = 2(6r + 7)(2r - 3)</math>, either <math>r = -\frac{7}{6}</math> or <math>r = \frac{3}{2}</math>. By the Rational Root Theorem, <math>-\frac{7}{6}</math> is not a root of <math>-16r^3 + 4r^2 + 45</math>. However, <math>-16\left(\frac{3}{2}\right)^3 + 4\left(\frac{3}{2}\right)^2 + 45 = -16 \cdot \frac{27}{8} + 4 \cdot \frac{9}{4} + 45 = -54 + 9 + 45 = 0</math>, so <math>\frac{3}{2}</math> is a root of <math>-16r^3 + 4r^2 + 45</math>. | ||
+ | |||
+ | Therefore, <math>r = \frac{3}{2} = 1.5</math>, so <math>\boxed{(\textbf{D})\ 1.52}</math> is closest to <math>r</math>. | ||
+ | |||
+ | -j314andrews | ||
+ | |||
+ | == Solution 3 == | ||
+ | |||
+ | Let <math>y = 2x</math>. Then <math>x = \frac{y}{2}</math>, so <math>8x^3 - 4x^2 - 42x + 45 = y^3 - y^2 - 21y + 45</math>. | ||
+ | |||
+ | By the Rational Root Theorem, any rational root of <math>y^3 - y^2 - 21y + 45</math> is a factor of <math>45</math>. Of the factors of <math>45</math>, <math>3</math> and <math>5</math> are roots, and <math>y^3 - y^2 - 21y + 45 = (y-3)^2(y-5)</math>. So <math>8x^3 - 4x^2 - 42x + 45 = (2x-3)^2(2x-5) = 8\left(x-\frac{3}{2}\right)^2\left(x-\frac{5}{2}\right)</math>, and <math>r = \frac{3}{2} = 1.5</math>. So <math>\boxed{(\textbf{D})\ 1.52}</math> is closest to <math>r</math>. | ||
+ | |||
+ | -j314andrews | ||
== See also == | == See also == |
Latest revision as of 19:13, 16 August 2025
Problem
For some real number , the polynomial
is divisible by
. Which of the following numbers is closest to
?
Solution 1
Since is a factor of
,
is a double root. Let
be the third root of
.
By Vieta's formulas, , so
.
Also, by Vieta's formulas, . Substituting
yields
. So
, that is,
. Therefore,
is either
or
.
Finally, by Vieta's formulas, . Though
does not satisfy this equation,
does.
Therefore, , so
is closest to
.
-e_power_pi_times_i, edited by j314andrews
Solution 2
By polynomial long division, .
So , that is,
is a root of both
and
.
Since , either
or
. By the Rational Root Theorem,
is not a root of
. However,
, so
is a root of
.
Therefore, , so
is closest to
.
-j314andrews
Solution 3
Let . Then
, so
.
By the Rational Root Theorem, any rational root of is a factor of
. Of the factors of
,
and
are roots, and
. So
, and
. So
is closest to
.
-j314andrews
See also
1980 AHSME (Problems • Answer Key • Resources) | ||
Preceded by Problem 23 |
Followed by Problem 25 | |
1 • 2 • 3 • 4 • 5 • 6 • 7 • 8 • 9 • 10 • 11 • 12 • 13 • 14 • 15 • 16 • 17 • 18 • 19 • 20 • 21 • 22 • 23 • 24 • 25 • 26 • 27 • 28 • 29 • 30 | ||
All AHSME Problems and Solutions |
These problems are copyrighted © by the Mathematical Association of America, as part of the American Mathematics Competitions.